Advertisement
If you have a new account but are having problems posting or verifying your account, please email us on hello@boards.ie for help. Thanks :)
Hello all! Please ensure that you are posting a new thread or question in the appropriate forum. The Feedback forum is overwhelmed with questions that are having to be moved elsewhere. If you need help to verify your account contact hello@boards.ie

Hmmmm... Physics Paper, how was it for you?

Options
12467

Comments

  • Registered Users Posts: 81 ✭✭Adventure


    Q8 on magnetism


    (i)0.01 sec
    (ii)0.02 Wb
    (iii)2 v
    Anyone get these??

    The question seemed a bit to easy.....:confused:



  • Registered Users Posts: 663 ✭✭✭SimpleLogic


    12a question with pole-vaulting guy.

    First part: 32.3?
    Second part: 5.418?


  • Closed Accounts Posts: 39 m-i-t-s-u-k-o


    muffinman wrote: »
    Don't have the paper but I think it was as simple as (120)(Wattage)(.96) but I'm not sure..

    I got 120,960J..
    Find the power output and then use the P= W/t formula


  • Closed Accounts Posts: 773 ✭✭✭Cokehead Mother


    Adventure wrote: »
    Q8 on magnetism


    (i)0.01 sec
    (ii)0.02 Wb
    (iii)2 v
    Anyone get these??

    The question seemed a bit to easy.....:confused:


    Yep. Agree with all 3.


  • Registered Users Posts: 140 ✭✭Slashman X


    Burner- wrote: »
    12a question with pole-vaulting guy.

    First part: 32.3?
    Second part: 5.418?
    1st - Yep
    2nd - Dont you take away the 1.1?


  • Advertisement
  • Registered Users Posts: 1,583 ✭✭✭alan4cult


    Burner- wrote: »
    12a question with pole-vaulting guy.

    First part: 32.3?
    Second part: 5.418?
    Ya spot on.


  • Registered Users Posts: 784 ✭✭✭Peleus


    Hm, I got 14 pions... to the people who got 7, are you certain you took into account that both protons had kinetic energy of one gigajoule (or whatever it was)?
    That or I did it totally wrong...
    I got 14. could have been wrong. Did you take the speed of light to be 3E8? it was supposed to be 2.9979E8. hopefully will only lose a mark for that.

    Also, when it said about the range of the fundamental forces.. did you need exact numbers? I'd never learned them.. I mean, I knew infinite for two of them, but for the other two I just said "tiny: within the nucleus" or whatever...

    No, i said strong ---> very small range (less than diameter of nucleus)
    and weak --->small range (less than diameter of atom) weak one is wrong i think.

    Average emf induced, off the top of my head, was 2 volts.

    And yeah got 15 sunrises.

    For deflecting a beam of electrons, whoever asked, I just said an electric or magnetic field... I guess a nucleus could technically too...

    I got all this. All right i think.


  • Closed Accounts Posts: 773 ✭✭✭Cokehead Mother


    Burner- wrote: »
    12a question with pole-vaulting guy.

    First part: 32.3?
    Second part: 5.418?

    I think I got 32.2 and something like 5 and a half.


  • Closed Accounts Posts: 121 ✭✭Patrickisperfec


    Physics was by far the hrdest exam


  • Registered Users Posts: 81 ✭✭Adventure


    Burner- wrote: »
    12a question with pole-vaulting guy.

    First part: 32.3?
    Second part: 5.418?

    got (i)32.2m
    (ii)5.8m


  • Advertisement
  • Registered Users Posts: 663 ✭✭✭SimpleLogic


    Slashman X wrote: »
    1st - Yep
    2nd - Dont you take away the 1.1?

    yeah you right damn :-)
    would i loose 3 marks for that?


  • Closed Accounts Posts: 773 ✭✭✭Cokehead Mother


    Burner- wrote: »
    yeah you right damn :-)
    would i loose 3 marks for that?

    I can't think of a reason why you would take away 1.1 from that answer. :confused:


  • Registered Users Posts: 140 ✭✭Slashman X


    I can't think of a reason why you would take away 1.1 from that answer. :confused:
    Height above the C.O.G


  • Registered Users Posts: 784 ✭✭✭Peleus


    The Hitman wrote: »
    I got 20,000pa. Probably wrong as usual!!:)

    thats right! i got that. :D woo!

    I'm so pissed off, i used 3E8 for speed of light in q10. should have been 2.9979E8. how many marks would i lose for that???

    Okay, here are ANSWERS:
    Please can somebody go over them ticking what they got etc...... thanks!!!!!!


    Section A:
    Q1
    More accurate
    Split cork
    l against T^2 (i got the axes mixed up but my graph is still technically correct innit???)
    i got 9.47 m/s^2

    Q2
    -preheated water so that errors that exist when water cools to room temp will be cancelled out by the same errors when cooling to final temp.
    -so no water affected the results. and so that the ice melted evenly and not at a very slow rate.
    -(mass of cal + water + ice) - (mass of cal + water) = mass of ice
    - 20.35 degrees cent.
    -5450 J
    -318063 J/Kg

    Q4
    -diagram etc...
    -R against θ. I put R on X axis and θ on Y axis. Is this wrong??????
    -3.6Ω
    -1.9Ω
    -didnt use graph to explain it. didn't know how. anyone see the correction here? NOT linear

    Q5
    -wight of floating body=weight of fluid displaced
    -20000Pa
    -f α 1/l
    -8E-19N
    -holes and electrons, + and - ions
    -magnetic field, electric field
    -Geiger Muller Tube (sp?) detects ionization in gas
    -not reactive, very small mass

    Q6
    - newtons law...
    -centripetal force (gravity), towards center of earth
    -g=8.564m/s^2
    -he's moving in a circle. Force towards center=body's desire to travel in straight line (bullsh1t)
    -Kepler's 3rd. T=5600s
    -its period is not =24hours
    -15 times

    Q8
    -def.
    -induced current opposes magnets motion. amplitude decreases
    -electrical--->Kinetic (wrong i think)
    -0.01s
    -0.02Wb
    -2V

    Q10a
    -definitions etc...
    -U,T,S (wrong, answer is C,U,T)
    -baryons (3 quarks) mesons (1q 1anti-q)
    -uud
    -2.53E-10 J
    -14 pions (wrong)

    Q11
    -373.15 K
    -def...
    -change in momentum = 5.4 kg m/s
    -i said a,b,gamma. but it was gamma, xray, infared etc...
    -def...
    -proved existance of atoms, allowed for more discoveries and future discoveries to be made. (I should have mentioned light being particles and waves)
    -2E-19 Joules
    -1.111E-9 Kg


  • Closed Accounts Posts: 7 lawdeedaw


    Peleus wrote: »
    I got 14. It was wrong tho sadly. Did you take the speed of light to be 3E8? it was supposed to be 2.9979E8. hopefully will only lose a mark for that.



    yep i got 14 aswel...my teacher done it out after and he got 14 aswel!!!:D:D

    thought it was a hard enough paper...i loved question 10a, 11 and 12!!!:D:D


  • Registered Users Posts: 1,583 ✭✭✭alan4cult


    No the right answer is 5.418

    Method 1:
    Consider man as a bullet upwards
    u=9.2
    v=0
    a=-9.8

    s= 4.318

    Now that tells you how much he has risen.
    And his centre of gravity is 1.1m above ground so you get 4.318 +1.1 = 5.418

    Methid 2:
    Treat it like energy
    Potential energy at start(taking ground as Pe=0)
    = mgh = 1.1mg
    Kinetic energy at start = 1/2m(9.2)^2 = 42.32m

    Potential Energy at end = mgh
    Ke = 0

    1.1mg + 42.32m = mgh
    1.1g + 42.32 = gh
    1.1 + 4.318 = h (Divide across by g)
    h = 5.418


  • Closed Accounts Posts: 773 ✭✭✭Cokehead Mother


    Slashman X wrote: »
    Height above the C.O.G

    I don't follow.

    Energy before = energy after

    P.E + K.E = P.E + K.E

    mgh1 + 1/2mv^2 = mgh2 + 0

    9.8(1.1) + 1/2 (9.2)^2 = (9.8)h

    h = 5.42

    Is there anything wrong with that?


  • Registered Users Posts: 252 ✭✭meeka


    I thought that was alright.. I hope!

    I got 11 pions for qt.10, I guess I forgot to add the original 3 pions, if that's what you had to do? Otherwise maybe I just completely screwed that up, it certainly felt like I was doing that question badly, anyway :P I did it out once, realised I was using 3x10^8 instead of 2.9979 or whatever it was, so I scribbled it out and did it again in the last 10 minutes. Hmmm.

    I did an extra long question though, so hopefully I managed to do okay, it's kind of hard to tell. I thought the experiments were grand, anyway! Thought the electromagnetic induction question was unusually easy.


  • Closed Accounts Posts: 773 ✭✭✭Cokehead Mother


    Shít you're right Meeka. Got 11 too. So everything was fine only I didn't add the first three? Feck feck feck. Stupid mistake. I also used 3E8 but it seems a lot of people did... maybe they'll be nice about it? :o


  • Closed Accounts Posts: 39 m-i-t-s-u-k-o


    Peleus wrote: »
    Q1
    More accurate
    Split cork
    l against T^2 (i got the axes mixed up but my graph is still technically correct innit???)
    i got 9.47 m/s^2
    T^2 on x-axis and l on y-axis is right, If you got them the wrong way, it changes your value for g if you worked it out from the grapgh.
    Peleus wrote: »
    Q2
    -preheated water so that errors that exist when water cools to room temp will be cancelled out by the same errors when cooling to final temp.
    -so no water affected the results. and so that the ice melted evenly and not at a very slow rate.
    -(mass of cal + water + ice) - (mass of cal + water) = mass of ice
    - 20.35 degrees cent.
    -5450 J
    -318063 J/Kg
    Yeps
    Peleus wrote: »
    Q4
    -diagram etc...
    -R against θ. I put R on X axis and θ on Y axis. Is this wrong??????
    -3.6Ω
    -1.9Ω
    -didnt use graph to explain it. didn't know how. anyone see the correction here? NOT linear
    I completely messed up this on but resistance on y-axis and temp on X-axis
    Peleus wrote: »
    Q5
    -wight of floating body=weight of fluid displaced
    -20000Pa
    -f α 1/l
    -8E-19N
    -holes and electrons, + and - ions
    -magnetic field, electric field
    -Geiger Muller Tube (sp?) detects ionization in gas
    -not reactive, very small mass
    Yar
    Peleus wrote: »
    Q6
    - newtons law...
    -centripetal force (gravity), towards center of earth
    -g=8.564m/s^2
    -he's moving in a circle. Force towards center=body's desire to travel in straight line (bullsh1t)
    -Kepler's 3rd. T=5600s
    -its period is not =24hours
    -15 times
    Weightless because W=mg and because the value for g is smaller his weight is reduced
    Got something about the same for the orbit
    Rest is the same


    Didn't do any of the others


  • Advertisement
  • Registered Users Posts: 26 Ally_ally


    nyangnyang wrote: »
    I am in shock. I learned 26 of the 28 experiments. The 2 i didnt learn came up.Anyone able to work out the probability of that(im serious i wanna know)
    i'm the exact same!!!!! it's sooooo annoying! that stupid q1 and q3...i attempted 1 but graph was all wrong...id say i got about 3 out of 4o...seriously!


  • Registered Users Posts: 81 ✭✭Adventure


    For the energy conversion in q8 ,

    Kinetic to Potential(since it slows down and is now hanging on a string):confused:


  • Registered Users Posts: 26 Ally_ally


    It was an awwwwwful paper! what was with the short questions in q5??? and the doppler q in 12 was really unclear...was it supossed to be the actual frequency or the apparent frequency. option a was hard too, and couldnt do 1 of the experiments. i always got As in physics, i'm definetely not going to get more than a b3 in this stupid paper. it's not fair :(


  • Registered Users Posts: 5,851 ✭✭✭PurpleFistMixer


    Hm, I said the conversion was kinetic to electric, as its kinetic energy is converted to the emf in the copper plate or whatever it was.

    Tis done now anyway.


  • Closed Accounts Posts: 3 YellowRabbit


    For the how many times does the astronaut see the sunrise thing, i got 15.4, but surely as .4 of a day is up until 9'oclock, he still would have seen the sixteenth sunrise even if it's not a whole number?

    Overall I thought the paper was fair enough.

    I got 14 pions too.


  • Closed Accounts Posts: 40 Ck1989


    For the thumb tack one i got 20Pa cos it's not as simple as just dividing mm^2 by 1000 to get m^2 -it's different like

    and i got 7 pions


  • Registered Users Posts: 1,583 ✭✭✭alan4cult


    For the how many times does the astronaut see the sunrise thing, i got 15.4, but surely as .4 of a day is up until 9'oclock, he still would have seen the sixteenth sunrise even if it's not a whole number?

    Overall I thought the paper was fair enough.

    I got 14 pions too.
    Thats if he started counting at midnight right?


  • Registered Users Posts: 1,583 ✭✭✭alan4cult


    For thumbtack question
    Pressure = Force/Area

    Area is 1666 2/3 times smaller
    Therefore Pressure is 1666 2/3 time greater

    Pressure = 12 * 1666 2/3 = 20000Pa


  • Closed Accounts Posts: 64 ✭✭Calorimeterman


    Yuk, looking at all you're answers I think that I did worse than I first did... I did an extra experiment question and an extra long question, but seeing as I had to leave out such a large sections of the course I didn't have much choice...

    I did the best I could have done though... so I have absolutely no regrets...


  • Advertisement
  • Closed Accounts Posts: 3 YellowRabbit


    alan4cult wrote: »
    Thats if he started counting at midnight right?

    Yeah, I guess it depends on whether he saw a sunset right away at the start of his orbit, so both answers could be right?


Advertisement